Is the sum of two negligible sets always a negligible set?












0












$begingroup$


Let $E$ and $F$ be two negligible sets, is the following sum always a negligible set?



$E + F = {x + y : x ∈
E, y ∈ F}$



I tend to say no but I can't find any counter exemple










share|cite|improve this question









$endgroup$

















    0












    $begingroup$


    Let $E$ and $F$ be two negligible sets, is the following sum always a negligible set?



    $E + F = {x + y : x ∈
    E, y ∈ F}$



    I tend to say no but I can't find any counter exemple










    share|cite|improve this question









    $endgroup$















      0












      0








      0





      $begingroup$


      Let $E$ and $F$ be two negligible sets, is the following sum always a negligible set?



      $E + F = {x + y : x ∈
      E, y ∈ F}$



      I tend to say no but I can't find any counter exemple










      share|cite|improve this question









      $endgroup$




      Let $E$ and $F$ be two negligible sets, is the following sum always a negligible set?



      $E + F = {x + y : x ∈
      E, y ∈ F}$



      I tend to say no but I can't find any counter exemple







      real-analysis measure-theory






      share|cite|improve this question













      share|cite|improve this question











      share|cite|improve this question




      share|cite|improve this question










      asked Jan 8 at 15:36









      Jonathan BaramJonathan Baram

      140113




      140113






















          1 Answer
          1






          active

          oldest

          votes


















          1












          $begingroup$

          In general no. For instance, if $C$ is the Cantor set, then $C+C=[0,2]$.






          share|cite|improve this answer









          $endgroup$









          • 2




            $begingroup$
            Thank you, I also found that the sum of the x-axis and the y-axis is equal to the whole plane for a more simple example
            $endgroup$
            – Jonathan Baram
            Jan 8 at 15:43











          Your Answer





          StackExchange.ifUsing("editor", function () {
          return StackExchange.using("mathjaxEditing", function () {
          StackExchange.MarkdownEditor.creationCallbacks.add(function (editor, postfix) {
          StackExchange.mathjaxEditing.prepareWmdForMathJax(editor, postfix, [["$", "$"], ["\\(","\\)"]]);
          });
          });
          }, "mathjax-editing");

          StackExchange.ready(function() {
          var channelOptions = {
          tags: "".split(" "),
          id: "69"
          };
          initTagRenderer("".split(" "), "".split(" "), channelOptions);

          StackExchange.using("externalEditor", function() {
          // Have to fire editor after snippets, if snippets enabled
          if (StackExchange.settings.snippets.snippetsEnabled) {
          StackExchange.using("snippets", function() {
          createEditor();
          });
          }
          else {
          createEditor();
          }
          });

          function createEditor() {
          StackExchange.prepareEditor({
          heartbeatType: 'answer',
          autoActivateHeartbeat: false,
          convertImagesToLinks: true,
          noModals: true,
          showLowRepImageUploadWarning: true,
          reputationToPostImages: 10,
          bindNavPrevention: true,
          postfix: "",
          imageUploader: {
          brandingHtml: "Powered by u003ca class="icon-imgur-white" href="https://imgur.com/"u003eu003c/au003e",
          contentPolicyHtml: "User contributions licensed under u003ca href="https://creativecommons.org/licenses/by-sa/3.0/"u003ecc by-sa 3.0 with attribution requiredu003c/au003e u003ca href="https://stackoverflow.com/legal/content-policy"u003e(content policy)u003c/au003e",
          allowUrls: true
          },
          noCode: true, onDemand: true,
          discardSelector: ".discard-answer"
          ,immediatelyShowMarkdownHelp:true
          });


          }
          });














          draft saved

          draft discarded


















          StackExchange.ready(
          function () {
          StackExchange.openid.initPostLogin('.new-post-login', 'https%3a%2f%2fmath.stackexchange.com%2fquestions%2f3066317%2fis-the-sum-of-two-negligible-sets-always-a-negligible-set%23new-answer', 'question_page');
          }
          );

          Post as a guest















          Required, but never shown

























          1 Answer
          1






          active

          oldest

          votes








          1 Answer
          1






          active

          oldest

          votes









          active

          oldest

          votes






          active

          oldest

          votes









          1












          $begingroup$

          In general no. For instance, if $C$ is the Cantor set, then $C+C=[0,2]$.






          share|cite|improve this answer









          $endgroup$









          • 2




            $begingroup$
            Thank you, I also found that the sum of the x-axis and the y-axis is equal to the whole plane for a more simple example
            $endgroup$
            – Jonathan Baram
            Jan 8 at 15:43
















          1












          $begingroup$

          In general no. For instance, if $C$ is the Cantor set, then $C+C=[0,2]$.






          share|cite|improve this answer









          $endgroup$









          • 2




            $begingroup$
            Thank you, I also found that the sum of the x-axis and the y-axis is equal to the whole plane for a more simple example
            $endgroup$
            – Jonathan Baram
            Jan 8 at 15:43














          1












          1








          1





          $begingroup$

          In general no. For instance, if $C$ is the Cantor set, then $C+C=[0,2]$.






          share|cite|improve this answer









          $endgroup$



          In general no. For instance, if $C$ is the Cantor set, then $C+C=[0,2]$.







          share|cite|improve this answer












          share|cite|improve this answer



          share|cite|improve this answer










          answered Jan 8 at 15:39









          José Carlos SantosJosé Carlos Santos

          157k22126227




          157k22126227








          • 2




            $begingroup$
            Thank you, I also found that the sum of the x-axis and the y-axis is equal to the whole plane for a more simple example
            $endgroup$
            – Jonathan Baram
            Jan 8 at 15:43














          • 2




            $begingroup$
            Thank you, I also found that the sum of the x-axis and the y-axis is equal to the whole plane for a more simple example
            $endgroup$
            – Jonathan Baram
            Jan 8 at 15:43








          2




          2




          $begingroup$
          Thank you, I also found that the sum of the x-axis and the y-axis is equal to the whole plane for a more simple example
          $endgroup$
          – Jonathan Baram
          Jan 8 at 15:43




          $begingroup$
          Thank you, I also found that the sum of the x-axis and the y-axis is equal to the whole plane for a more simple example
          $endgroup$
          – Jonathan Baram
          Jan 8 at 15:43


















          draft saved

          draft discarded




















































          Thanks for contributing an answer to Mathematics Stack Exchange!


          • Please be sure to answer the question. Provide details and share your research!

          But avoid



          • Asking for help, clarification, or responding to other answers.

          • Making statements based on opinion; back them up with references or personal experience.


          Use MathJax to format equations. MathJax reference.


          To learn more, see our tips on writing great answers.




          draft saved


          draft discarded














          StackExchange.ready(
          function () {
          StackExchange.openid.initPostLogin('.new-post-login', 'https%3a%2f%2fmath.stackexchange.com%2fquestions%2f3066317%2fis-the-sum-of-two-negligible-sets-always-a-negligible-set%23new-answer', 'question_page');
          }
          );

          Post as a guest















          Required, but never shown





















































          Required, but never shown














          Required, but never shown












          Required, but never shown







          Required, but never shown

































          Required, but never shown














          Required, but never shown












          Required, but never shown







          Required, but never shown







          Popular posts from this blog

          Can a sorcerer learn a 5th-level spell early by creating spell slots using the Font of Magic feature?

          Does disintegrating a polymorphed enemy still kill it after the 2018 errata?

          A Topological Invariant for $pi_3(U(n))$